[Help] Metastock Formula Explorer

#1
anyone can write an explorer formula for me?
my requirements are :

- it's price currently 4%-5% below it's all time high price
- today's volume > yesterday volume > the day before yest. volume

and can someone give me explorer formula that'll be useful for darvas trading style?

thanks in advance
 

oxusmorouz

Well-Known Member
#2
anyone can write an explorer formula for me?
my requirements are :

- it's price currently 4%-5% below it's all time high price
- today's volume > yesterday volume > the day before yest. volume

and can someone give me explorer formula that'll be useful for darvas trading style?

thanks in advance
Column A: {Condition}
a:= Highest(c);
b:= C < .96*a And C > .95*a;
d:= V > Max(Ref(V,-1),Ref(V,-2)) And Ref(V,-1) > Ref(V,-2);
b and d;

Filter:
ColA

x-------------------------------------------x
 
#4
Column A: {Condition}
a:= Highest(c);
b:= C < .96*a And C > .95*a;
d:= V > Max(Ref(V,-1),Ref(V,-2)) And Ref(V,-1) > Ref(V,-2);
b and d;

Filter:
ColA

x-------------------------------------------x
many thx oxusmorouz...

how about if i only want to search all stock that it's currently closing price between it's all time high and 2% below the all time high.
I've tried like this, but the outcome wasn't i wanted, too many results.

Column A: {Condition}

Close >= .98*Highest(price);

Filter:
ColA

please correct me...
THX

newbie
 

oxusmorouz

Well-Known Member
#5
many thx oxusmorouz...

how about if i only want to search all stock that it's currently closing price between it's all time high and 2% below the all time high.
I've tried like this, but the outcome wasn't i wanted, too many results.

Column A: {Condition}

Close >= .98*Highest(price);

Filter:
ColA

please correct me...
THX

newbie
Column A: {Condition}

Close >= .98*Highest(C);

Filter:
ColA

Your coding seems correct. Not sure why the results are inconsistent.
 
#6
hi
can any body tell me how can i draw a chart for A/b, where A and B are two different stocks. I found that there is a option for drawing the chart for A-B but could not figure out for A/B.
Regards
Anup
 

Similar threads